the mayor of a town has proposed a plan for the construction of a new community. a political study took a sample of 1100 voters in the town and found that 25% of the residents favored construction. using the data, a political strategist wants to test the claim that the percentage of residents who favor construction is more than 22%. determine the decision rule for rejecting the null hypothesis, h0, at the 0.01 level.

Answers

Answer 1

a) Null Hypothesis: H₀: P>0.22

Alternative hypothesis: H₁:P<0.22

b) Test statistic  Z = 2.5

The sample size is n = 1100.

Given the information, political research selected 1100 voters from the town and discovered that 25% of the citizens supported the building.

Given sample percentage "p" = 25% = 0.25, this follows.

A campaign strategist decided to check the assertion that more than 22% of people support building based on available data.

Assuming Population Proportion 'P' = 22% = 0.22

Q = 1 - p = 1 - 0.22=0.78

Given the available information, a political strategist wishes to test the assertion that more citizens prefer building than 22%, thus we assume the null hypothesis is true.

Null Hypothesis: H₀: P > 0.22

Alternative hypothesis: H₁: P < 0.22

The test of statistic

Z = (p - P) ÷ √(PQ ÷ n)

Z = (0.25 - 0.22) ÷ √(0.22 × 0.78 ÷ 1100)

Z = 2.5

Learn more about the null hypothesis at

https://brainly.com/question/23693764?referrer=searchResults

#SPJ4


Related Questions

0.006 Reapting in a fraction in simplest form ​

Answers

Written as a fraction 3/50

Answer:

3/500

Step-by-step explanation:

6/1000 ÷2 =

3/500

if zeros are reciprocal of each other then find k. (1-3k)x2 +4x-5

Answers

Answer: Let's assume that the zeros of the given quadratic equation are a and 1/a, since they are reciprocals of each other. By the sum-product relationships for zeros of a quadratic equation, we have:

a + 1/a = -b/a1 = -4/(1-3k) (where b = 4 and a1 = 1-3k)

Multiplying both sides by a1, we get:

a1(a + 1/a) = -4

Expanding the left-hand side, we get:

a1a + a1/a = -4

Multiplying both sides by a, we get:

a1a² + a1 = -4a

Substituting a1 = 1-3k, we get:

(1-3k)a² + (1-3k) = -4a

Rearranging, we get:

(1-3k)a² + 4a + (3k-1) = 0

Since a is one of the zeros of the quadratic, we know that the quadratic can be factored as:

(1-3k)(a - r)(a - s) = 0

where r and s are the other two roots of the quadratic. By expanding the left-hand side, we get:

(1-3k)(a² - (r+s)a + rs) = 0

Comparing with the expanded form of the quadratic, we see that:

r + s = -4/(1-3k)

rs = (3k-1)/(1-3k)

By Vieta's formulas, we know that rs = c/a1, where c is the constant term of the quadratic. Substituting c = -5 and a1 = 1-3k, we get:

rs = -5/(1-3k)

Equating this with the expression we obtained earlier for rs, we get:

-5/(1-3k) = (3k-1)/(1-3k)

Multiplying both sides by 1-3k and simplifying, we get:

-5 = (3k-1)(3k-2)

Expanding the right-hand side, we get:

-5 = 9k² - 15k + 2

Simplifying, we get:

9k² - 15k - 7 = 0

Using the quadratic formula, we get:

k = (15 ± sqrt(15² + 497))/18

k = (15 ± sqrt(429))/18

Therefore, the two possible values of k are:

k = (15 + sqrt(429))/18 ≈ 1.57

k = (15 - sqrt(429))/18 ≈ -0.24

Note that both of these values of k satisfy the condition that the zeros of the quadratic are reciprocals of each other.

Step-by-step explanation:

HELP MEEE!! What's the domain and range?

HELP MEEE!! What's the domain and range?

Answers

Answer:

domain: [- ∞ , ∞]

range: (8 , -∞]

Step-by-step explanation:

- ∞ < x < ∞

y < 8

Please help!! If you can.

Please help!! If you can.

Answers

Answer:

4 is the answer is this equation

Graph the line with the equation y = 4/5x + 2

Answers

Answer:  I have provided a screenshot of what your graph should look like so uh yeah you got this!

Graph the line with the equation y = 4/5x + 2

The graph of the line with the equation y = 4/5x + 2 is given in below attachment.

To graph the line with the equation y = (4/5)x + 2, we can use the slope-intercept form, which is y = mx + b.

where "m" represents the slope and "b" represents the y-intercept.

In this case, the slope (m) is 4/5, and the y-intercept (b) is 2.

To plot the line, we can start by plotting the y-intercept, which is the point (0, 2).

From there, we can use the slope to find additional points and connect them to form the line.

Let's plot a few more points:

When x = 5:

y = (4/5)(5) + 2

= 4 + 2 = 6

So we have the point (5, 6).

When x = -5:

y = (4/5)(-5) + 2

= -4 + 2

= -2

So we have the point (-5, -2).

To learn more on slope intercept form click:

https://brainly.com/question/9682526

#SPJ6

Graph the line with the equation y = 4/5x + 2

h(t) = t + 2 + 3; Find h(6)

Answers

If you wanted a step by step:

6 = t in this case so simply plug in 6 to the equation and get 6 + 2 + 3, after adding you’ll have 11.

Answer: h(6) = 11
Answer:

h(6) = 11

Step-by-step explanation:

Hi there !

h(t) = t + 2 + 3

= t + 5

replace t = 6

h(6) = 6 + 5

= 11

Good luck !


Please help em fast I really need help


lf ƒ(x) = 2(x + 1)² and g(x) = 3x- 2 determine f[g(2)]

Answers

Answer:

f(g(2)) = 50

Step-by-step explanation:

evaluate g(2) then substitute the result obtained into f(x)

g(2) = 3(2) - 2 = 6 - 2 = 4 , then

f(4) = 2(4 + 1)² = 2(5)² = 2(25) = 50

Answer:

f[g(2)] = 50

Step-by-step explanation:

Given functions:

\(f(x)=2(x+1)^2\)

\(g(x)=3x-2\)

We are asked to determine f[g(2)], which is known as a composite function. When solving composite functions, you always work inside out.

Step 1: Find the value of g(2) by substituting 2 for x in function g(x).

\(\implies g(2)=3(2)-2\)

\(\\\implies g(2)=6-2\Rightarrow g(2)=\boxed{4}\)

Step 2: Substitute the found value into the composite function.

\(\\\implies f[g(2)]= 2(4+1)^2\)

\(\implies f[g(2)] = 2(5)^2 \Rightarrow 2(25) \Rightarrow \boxed{50}\)

Hence, the value of the composite function f[g(2)] is 50.

Learn more about composite functions here:

brainly.com/question/28062427

brainly.com/question/26215408

What information would verify that LMNO is an
isosceles trapezoid? Check all that apply.
a. LN MO
b. LM ON
c. LO = MN
d. ZL = ZN
e. ZLEM
f. IRR

Answers

The correct answers are options b. LM ≅ ON and d. ∠L ≅ ∠N as they verify that LMNO is an isosceles trapezoid.

To verify that LMNO is an isosceles trapezoid, we need to check if the following conditions are met:
1. One pair of opposite sides must be parallel (base angles are equal).
2. The non-parallel sides (legs) are congruent.
From the given options:
a. LN ≅ MO - This does not verify LMNO as an isosceles trapezoid, as it only shows congruence between the parallel sides.
b. LM ≅ ON - This verifies LMNO as an isosceles trapezoid, as it shows that the non-parallel sides (legs) are congruent.
c. LO ≅ MN - This does not verify LMNO as an isosceles trapezoid, as it only shows congruence between the diagonals.
d. ∠L ≅ ∠N - This verifies LMNO as an isosceles trapezoid, as it shows that the base angles are congruent, indicating that the opposite sides are parallel.
e. ∠L ≅ ∠M - This does not verify LMNO as an isosceles trapezoid, as it only shows congruence between adjacent angles.
Thus, options b and d are the correct answers that verify LMNO is an isosceles trapezoid.

Learn more about trapezoid here:

https://brainly.com/question/14458711

#SPJ11

Ejemplo: Martin necesita sembrar unos terrenos de cultivo. Ha gestionado un
préstamo del Banco "A" de S/ 5000 para pagar en tres años con una tasa de
interés del 4% mensual, ¿qué cantidad de interés pagará?
Datos: C= S/ 5000 t= 3 años r= 4% (mensual) = ?
8​

Answers

Answer:

Step-by-step explanation:

10

How long will an initial bank deposit of $10,000 grow to $23,750 at 5% annual compound interest?

Answers

Given:

P = $10000

r= 5%

Find:

we have to find the time for which initial bank deposit of $10,000 grow to $23,750 at 5% annual compound interest.

Explanation:

The formula for compound interest is

First, convert R as a percent to r as a decimal

r = R/100

r = 5/100

r = 0.05 per year,

Then, solve the equation for t

t = ln(A/P) / n[ln(1 + r/n)]

t = ln(23,750.00/10,000.00) / ( 1 × [ln(1 + 0.05/1)] )

t = ln(23,750.00/10,000.00) / ( 1 × [ln(1 + 0.05)] )

t = 17.729 years

Summary:

The time required to get a total amount of $23,750.00 with compounded interest on a principal of $10,000.00 at an interest rate of 5% per year and compounded 1 times per year is 17.729 years.

(about 17 years 9 months)

use analytic methods to find (a) the local ex- trema, (b) the intervals on which the function is increasing, and (c) the intervals on which the function is decreasing

Answers

Use analytic methods,

(a) The local extrema,

1) If f'(x) > 0 for all x on (a , c) and f'(x)<0 for all x on (c , b), then f(c) is a local maximum value.

2) If f'(x) < 0 for all x on (a , c) and f'(x)>0 for all x on (c , b), then f(c) is a local maximum value.

(b) The intervals on which the function is increasing

Write properties of function :

Increasing interval : ( - ∞ , 0 )

(c) The intervals on which the function is decreasing

Write properties of function :

Decreasing interval : ( 0 , ∞ )

Given that,

Use analytic methods (a) the local extrema, (b) the intervals on which the function is increasing, and (c) the intervals, then the function is,

A function's growing (or decreasing) periods match the periods when its derivative is positive (or negative). As a result, we can easily determine the intervals where a function increases or decreases by taking its derivative and analyzing it to determine if it is positive or negative.

(a) How do we find the local extrema?

Let f be continuous on an open interval (a , b) that contains a critical x-value.

1) If f'(x) > 0 for all x on (a , c) and f'(x)<0 for all x on (c , b), then f(c) is a local maximum value.

2) If f'(x) < 0 for all x on (a , c) and f'(x)>0 for all x on (c , b), then f(c) is a local maximum value.

(b) The intervals on which the function is increasing

Write properties of function :

Increasing interval : ( - ∞ , 0 )

(c) The intervals on which the function is decreasing

Write properties of function :

Decreasing interval : ( 0 , ∞ )

Therefore,

Use analytic methods,

(a) The local extrema,

1) If f'(x) > 0 for all x on (a , c) and f'(x)<0 for all x on (c , b), then f(c) is a local maximum value.

2) If f'(x) < 0 for all x on (a , c) and f'(x)>0 for all x on (c , b), then f(c) is a local maximum value.

(b) The intervals on which the function is increasing

Write properties of function :

Increasing interval : ( - ∞ , 0 )

(c) The intervals on which the function is decreasing

Write properties of function :

Decreasing interval : ( 0 , ∞ )

To learn more about Analytic methods visit :

brainly.com/question/19463434

#SPJ4

I NEED HELP GUYS PLEASE I CANT FAIL AGAIN A translation was applied to square ABCD to form
A'B'C'D.
Which line segment defines the translation?
SHE
ОАС!
y
5
OBD
А
B
4.
O C'B
3
O DA
2-
D
A'
C
B'
-5-4-3-
2
3
4
5
X
D
C

I NEED HELP GUYS PLEASE I CANT FAIL AGAIN A translation was applied to square ABCD to formA'B'C'D.Which

Answers

B is the right answer
The correct answer is: B

No trolls or links or i will report

No trolls or links or i will report

Answers

Answer:

92.4 ft²

Step-by-step explanation:

Total surface area comprises one rectangular base and two pairs of diferent triangles.

Area of rectangle

A = lw

Area of triangle

A = 1/2bh

Substitute and find

A = 7*4 + 2(1/2*4*6.3) + 2(1/2*7*5.6) = 92.4 ft²

Area of base rectangle

7(4)28ft²

Area of rest triangles

2(1/2BH)+2(1/2bh)BH+bh4(6.3)+7(5.6)64.56ft²

Now

TSA

64.56+2892.56ft²

help me..........................​

help me..........................

Answers

Answer:

d.90l

Step-by-step explanation:

Question 4 of 10
Which of the following could be the ratio between the lengths of the two legs
of a 30-60-90 triangle?
Check all that apply.
□A. √2:√2
B. 15
□ C. √√√√5
□ D. 12
DE √3:3
OF. √2:√5
←PREVIOUS
SUBMIT

Question 4 of 10Which of the following could be the ratio between the lengths of the two legsof a 30-60-90

Answers

The ratios that could be the lengths of the two legs in a 30-60-90 triangle are √3:3 (option E) and 12√3 (option D).

In a 30-60-90 triangle, the angles are in the ratio of 1:2:3. The sides of this triangle are in a specific ratio that is consistent for all triangles with these angles. Let's analyze the given options to determine which ones could be the ratio between the lengths of the two legs.

A. √2:√2

The ratio √2:√2 simplifies to 1:1, which is not the correct ratio for a 30-60-90 triangle. Therefore, option A is not applicable.

B. 15

This is a specific value and not a ratio. Therefore, option B is not applicable.

C. √√√√5

The expression √√√√5 is not a well-defined mathematical operation. Therefore, option C is not applicable.

D. 12√3

This is the correct ratio for a 30-60-90 triangle. The ratio of the longer leg to the shorter leg is √3:1, which simplifies to √3:3. Therefore, option D is applicable.

E. √3:3

This is the correct ratio for a 30-60-90 triangle. The ratio of the longer leg to the shorter leg is √3:1, which is equivalent to √3:3. Therefore, option E is applicable.

F. √2:√5

This ratio does not match the ratio of the sides in a 30-60-90 triangle. Therefore, option F is not applicable. So, the correct option is D. 1 √2.

For more such questions on lengths

https://brainly.com/question/28322552

#SPJ8

The table below shows that the number of miles driven by Jamal is directly proportional to the number of gallons he used.
Gallons Used
Gallons Used
Miles Driven
Miles Driven
14
14
525
525
43
43
1612.5
1612.5
47
47
1762.5
1762.5
How many gallons of gas would he need to travel
296.25
296.25 miles

Answers

Jamal would need approximately 7.9 gallons of gas to travel 296.25 miles.

We can use the concept of direct variation to solve this problem. Direct variation means that two quantities are related by a constant ratio. In this case, the number of miles driven is directly proportional to the number of gallons used.

To find the constant of proportionality, we can use the given data. From the table, we can see that when Jamal used 14 gallons, he drove 525 miles. So we can write:

14/525 = k

where k is the constant of proportionality.

Solving for k, we get:

k = 14/525

Now we can use this value of k to find how many gallons Jamal would need to travel 296.25 miles. Let x be the number of gallons he would need. Then we can write:

x/296.25 = k

Substituting the value of k, we get:

x/296.25 = 14/525

Solving for x, we get:

x = (296.25 × 14) / 525

x ≈ 7.9

To learn more about proportionality click on,

https://brainly.com/question/29005048

#SPJ1

For what value of x is the equations 2^2x+7 = 2^15 true?

Answers

Answer:

Step-by-step explanation:

To find the value of x, solve the equation. Do this by rearranging the equation to make x the subject.

Answer:

x=32761 /4

Step-by-step explanation:

Step 1: Simplify both sides of the equation.

4x+7=32768

Step 2: Subtract 7 from both sides.

4x+7−7=32768−7

4x=32761

Step 3: Divide both sides by 4.

What is 85 degrees Fahrenheit in Celsius?

Answers

Answer:

29.4444

Step-by-step explanation:

The answer is 29.444 degrees Celsius.

Find the x and y intercept of y=-3x+4

Answers

Answer:

I've attached the answer below.

Step-by-step explanation:

Find the x and y intercept of y=-3x+4

did it tell you what x and y equal

for what values of x in [0,2π] does the graph of f(x)=x+2sinx have a horizontal tangent?

Answers

the values of x in [0, 2π] where the graph of f(x) has a horizontal tangent are x = 2π/3, x = 4π/3, and x = π.

To find the values of x in [0, 2π] where the graph of f(x) = x + 2sin(x) has a horizontal tangent, we need to find where the derivative of the function is zero or undefined.

The derivative of f(x) is:

f'(x) = 1 + 2cos(x)

For the derivative to be zero, we need:

1 + 2cos(x) = 0

Solving for cos(x), we get:

cos(x) = -1/2

This is true when x = 2π/3 or x = 4π/3.

Now we need to check if the derivative is undefined at any point in the interval [0, 2π]. The derivative is undefined when cos(x) = -1, which occurs at x = π.

To know more about derivative visit:

brainly.com/question/29144258

#SPJ11

Which equation represents the graphed function?
A)-3x + 2 = y
B) -2/3x+2=y
C)3/2x-3=y
D)2x-3=y​

Which equation represents the graphed function? A)-3x + 2 = y B) -2/3x+2=yC)3/2x-3=yD)2x-3=y

Answers

Answer:

(C) 3/2x-3=y

Step-by-step explanation:

The point on the x-axis is 2

The point on the y - axis is -3

#3) Find the equation of the linear relationship represented in the table below.
x
у
-7.
5
-3
1
1
-3
5
-7

Answers

It’s alright the app is alright but it is a little bit more than a few other side effects and green beans in my life but I don’t think it’s alright you can start the game and start it

Brandon is not smiling. He just left the
dentist's office, and he has 5 cavities! If
it takes the dentist 14 minutes to fill one
cavity, how long will it take to fill all 5 of
Brandon's cavities?

Answers

Answer:

it should take 70 mins to fill all 5

hope this helps!! :)

Step-by-step explanation:

14 x 5 = 70

Answer:

70

Step-by-step explanation:

14 minutes = 1 cavity

x minutes = 5 cavities

70= x

Find the slope of the line graphed below.

Find the slope of the line graphed below.

Answers

Answer:

m = 3/4

Step-by-step explanation:

Start in (- 2,1) up three and four to the right. Positive slope. (Going uphill positive +, and downhill is negative - )

i need help with this

i need help with this

Answers

Since 12 is hypotenuse, formula we would use is:
12^2=5^2+x^2
144=25+x^2
x^2= 144-25
x^2=119
x= √119
x=10.9087
Rounded to nearest tenth,
x=10.9
Good luck.

which term describes the percent of voters casting ballots?

Answers

The term which is used for describing the number of voters who casted their votes using the ballots is called as voter turnout.

The process of selecting a candidate for a suitable post who takes care of the policies which will be framed for the people is called as voting. In this process, the members/ citizens casts their vote in the favor of their suitable candidate. Many times voters are not valid and their votes are considered as null and void.

The voter turnout is either the percentage of registered voters, eligible voters, or all voting-age people ( this varies from place to place depending upon the minimum voting age required). The voter turnout is also referred to the number of people who go to it or take part in it. High voter turnout is generally considered a sign of a healthy democracy.

Learn more about voter turnout at:

brainly.com/question/29639449

#SPJ4

One angle of a parallelogram measures 37°. What are the measures of the other three angles in the parallelogram?

Answers

The measures of the other two angles in the parallelogram are both 143 degrees.

what is a parallelogram?

A parallelogram is a quadrilateral with two sides that are parallel to one another. A parallelogram has equal-length opposite sides and equal-length opposite angles. The internal angles that are extra to the transversal on the same side. A parallelogram's total interior angles add up to 360 degrees.

A parallelogram is formed by the junction of two parallel lines, and its opposite angles are equal.

Then, there are two 35 degree angles. Two further angles that are equivalent are noted: A parallelogram's angles add up to 360 degrees. Following that, x+x+37+37=360

2x=360-74

2x=286

x=143.

To know more about parallelogram visit:

brainly.com/question/29147156

#SPJ1

 HELP ASAP TEST IS OVER SOON!!! PLEASEEE
A television screen is a rectangle 35 inches wide and 20 inches long. A television size is advertised as the length of its diagonal. What is the size of the television? In other words, what is the length of the diagonal? Around the answer to the nearest hundredth.

Answers

Answer:

35 in

Step-by-step explanation:

Answer: 435

Explanation: The square root of 35^2 + 20^2

Give an example of a function that is a dilation and a reflection of the parent function.

Answers

Answer: f(x) = -1/2(x+3)

Example: g(x) = -1/2(-x-3)

This function is a dilation and a reflection of the parent function f(x) because when graphed, the two functions have the same shape but are reflected over the y-axis and are half the size of the original.

Matthew wants to take out a loan to buy a car. He calculates that he can make repayments of $35,000 per year. If he can get a six-year loan with an interest rate of 9.25%, what is the maximum price he can pay for the car?

Answers

The maximum price Matthew can pay for the car, considering his repayment capability and the loan terms, is approximately $126,318.29.

To determine the maximum price Matthew can pay for the car, we need to consider his repayment capability and the terms of the loan.

Matthew can make annual repayments of $35,000. Since the loan term is six years, the total amount he can repay over the loan period is $35,000 multiplied by six, which equals $210,000.

To calculate the maximum price of the car, we need to account for the interest rate of 9.25%. The interest rate represents the cost of borrowing and is applied to the loan amount.

Let's assume the loan amount is denoted by P.

The formula to calculate the future value of a loan with interest is:

FV = P(1 + r)^n

Where:

FV = Future value (total amount repaid)

P = Principal amount (maximum price of the car)

r = Interest rate per period (9.25% or 0.0925)

n = Number of periods (six years)

Since Matthew can repay a total of $210,000 over the loan period, we can set up the equation:

$210,000 = P(1 + 0.0925)^6

Now we can solve for P:

P = $210,000 / (1 + 0.0925)^6

Evaluating this expression, we find:

P ≈ $126,318.29

Therefore, the maximum price Matthew can pay for the car, considering his repayment capability and the loan terms, is approximately $126,318.29.

Learn more about capability here

https://brainly.com/question/31154332

#SPJ11

Other Questions
Which of the following refers to payments an individual makes as a condition of receiving medical services?A copaymentsB deductibleC coinsuranceD premium Which of the following layers of rock is the oldest?Option 1: AOption 2: aOption 3: BOption 4: C 12 4/9 +14 5/12= Whats the answer for this problem Which statement describes one feature of Rutherfords model of the atom?Oppositely charged objects attract each other. This attraction holds electrons in atoms and holds atoms to one another in many compounds. However, Ernest Rutherfords model of the atom failed to explain why electrons were not pulled into the atomic nucleus by this attraction. in _____ britain withdrew from the european union following _____. Please help me with this assignment, dont put that link Tell me if you need any of my vocabulary sheet words a combination of longitudinal and transverse waves is called a Julien est journaliste. Lintonation ascendante : Julien est journaliste ? Question avec est-ce que : Est-ce que Julien est journaliste ? Linversion du sujet : Est-Julien journaliste ? Tu aimes le chocolat. Lintonation ascendante : Tu aimes le chocolat ? Question avec est-ce que : Est-ce que tu aimes le chocolat ? Linversion du sujet : Aimes-tu le chocolat ? Vous restez la maison. Lintonation ascendante : Vous restez la maison ? Question avec est-ce que : Est-ce que vous restez la maison ? Linversion du sujet : Restez-vous la maison ?Julien est journaliste. Lintonation ascendante : Julien est journaliste ? Question avec est-ce que : Est-ce que Julien est journaliste ? Linversion du sujet : Est-Julien journaliste ? Tu aimes le chocolat. Lintonation ascendante : Tu aimes le chocolat ? Question avec est-ce que : Est-ce que tu aimes le chocolat ? Linversion du sujet : Aimes-tu le chocolat ? Vous restez la maison. Lintonation ascendante : Vous restez la maison ? Question avec est-ce que : Est-ce que vous restez la maison ? Peux-tu vrifier? Merci all of the following are deposit-type institutions that serve as financial intermediaries except: multiple choice question. commercial banks savings and loans insurance companies credit unions On the spinner below, find the probability ofa number greater than 6 or a blue.Helppppp What is the example of formal credit? True or false: Education, the second E of the 4E framework, includes reminding people what they already know. ten physicians have just completed their residencies in internal medicine and are considering opening a group practice. they estimate the practice would have the following annual cost structure: you must show your calculation to receive credit. annual fixed costs $750,000 variable cost per visit $ 50 if volume in the first year is estimated to be 10,000 visits, what price per visit must be set if the practice wants to make an annual profit of $160,000? Are the tube feet at the tip of the arm the same as those further down the arm? why might this be? What occurs in the introduction of the story?a woman dies violently.Ba son and daughter listen to their mother's last words.two adults mourn the peaceful death of their mother.Dtwo siblings pray for their awful mother to be let into heaven. Help!!!! Can anyone answer this lab questions below about mitosis!!!! Can a sole proprietorship company become a limited liability company? PLEASE ANSWER ASAP Question 6 (2 points)Some chickens lay an egg every day. Other chickens produce few eggs. A study is planned to examine some factors that might affect the number of eggs produced by chickens. Which of the following is NOT a suitable hypothesis for the study?Question 6 options:If the cage for the chickens is larger, they will produce more eggs.If more eggs are produced by the chickens, they will lose more weight.If there is more protein in the feed, then more eggs are produced.If chickens receive more hours of light, then they will produce more eggs.Question 6 (2 points)Some chickens lay an egg every day. Other chickens produce few eggs. A study is planned to examine some factors that might affect the number of eggs produced by chickens. Which of the following is NOT a suitable hypothesis for the studyIf the cage for the chickens is larger, they will produce more eggs.If more eggs are produced by the chickens, they will lose more weight.If there is more protein in the feed, then more eggs are produced.If chickens receive more hours of light, then they will produce more eggs.Sam wished to investigate how fertilizer run-off affects the growth of algae in freshwater lakes and streams. He set his experiment up as follows. He placed 900 ml of water into each of five 1000 ml glass beakers. To each beaker, he added 5 ml of water from an aquarium which contained a large concentration of algae. The beakers were placed under a lamp which was on a timer to provide 12 hours of light each day. Liquid fertilizer was added to the beakers in the following amounts.Each week, a random sample from each of the beakers was examined under a microscope to get a count of the number of algal cells present.What is the independent variable in this experiment?Question 7 options:amount of fertilizernumber of algal cellshours of daylightamount of waterQuestion 8 (2 points)Sam wished to investigate how fertilizer run-off affects the growth of algae in freshwater lakes and streams. He set his experiment up as follows. He placed 900 ml of water into each of five 1000 ml glass beakers. To each beaker, he added 5 ml of water from an aquarium which contained a large concentration of algae. The beakers were placed under a lamp which was on a timer to provide 12 hours of light each day. Liquid fertilizer was added to the beakers in the following amounts.Each week, a random sample from each of the beakers was examined under a microscope to get a count of the number of algal cells present.What is the dependent variable in this experiment?Question 8 options:size of beakersnumber of algal cellsamount of fertilizerhours of daylightQuestion 9 (2 points)Sam wished to investigate how fertilizer run-off affects the growth of algae in freshwater lakes and streams. He set his experiment up as follows. He placed 900 ml of water into each of five 1000 ml glass beakers. To each beaker, he added 5 ml of water from an aquarium which contained a large concentration of algae. The beakers were placed under a lamp which was on a timer to provide 12 hours of light each day. Liquid fertilizer was added to the beakers in the following amounts.Each week, a random sample from each of the beakers was examined under a microscope to get a count of the number of algal cells present.Which of the following is the control group for the experiment?Question 9 options:Beaker DBeaker CBeaker EBeaker ABeaker BQuestion 10 (2 points)To focus a specimen, it is best to start with which objective:Question 10 options:High powerLow powerScanningOcularOn the image, which number represents the coarse adjustment knob?12510What important feature is missing from this line graph representing sea level change between 1870 And 1990X axis labela titleY axis labelappropriately spaced scales In a certain ecosystem, there is a population of lynx that feed on snowshoe hares. If there is less food for the hare for several years, what will most likely be the result for the lynx population? A. The lynx population will increase. B. The lynx population will decrease. C. The lynx population will go extinct. D. The lynx population will stay the same. please solve quickly and give solution before hand if possible